Math, asked by moreirawendy52, 1 month ago

guys please help its urgent

Attachments:

Answers

Answered by daschaitali047
1

Answer:

a. 40

Step-by-step explanation:

Please mark me as brainliest

Answered by Anonymous
5

Answer:

40° option a is the accurate answer

Similar questions